SU(3)SU(3)SU(3) irreduzible Darstellungen mit Tensormethode

Ich beschäftige mich mit der Tensorproduktdarstellung von S U ( 3 ) und ich habe einige Probleme, einige Zerlegungen zu verstehen.

1) Finden wir die irreduzible Darstellung von 3 3 ¯

wir haben, dass sich diese Darstellung wie transformiert

T ' J ich = U k ich U J l T l k

daher beobachte ich das

T R ( T ) = δ ich J T J ich
ist eine Invariante und so

T J ich = ( T J ich 1 3 δ ich J T J ich ) + 1 3 δ ich J T J ich

lässt mich schreiben

3 3 ¯ = 8 1
Hier kommen meine Fragen: Ich habe gehört, dass dies 8 Vertretung ist ein „ 8 M A " wobei MA für "gemischt-antisymmetrisch" steht. Die Bedeutung von "gemischt-antisymmetrisch" sollte sein: "der Tensor ( T J ich 1 3 δ ich J T J ich ) sollte für einen Austausch von 2 bestimmten Indizes antisymmetrisch sein, aber nicht für einen allgemeinen Austausch von 3 Indizes". Was bedeutet das? Ich sehe nur 2 Indizes in diesem Tensor.

2) Betrachten Sie diese Darstellung:

3 3 3 = 3 ( 6 3 ¯ ) = 3 6 S 3 3 ¯ = 3 6 S 8 M A 1

und jetzt auf meinen Notizen habe ich

3 6 S = 10 S 8 M S

Wobei "MS" für "gemischt symmetrisch" steht: symmetrisch für einen Austausch von 2 bestimmten Indizes, aber nicht für einen allgemeinen Austausch von 3 Indizes.

Ich konnte diese letzte Zerlegung mit der Tensormethode nicht demonstrieren. Ich fing an, das zu bemerken:

3 6 S = Q ich S k , l
Wo S k , l ist ein symmetrischer Tensor. Aber dann bin ich nicht in der Lage, die obige Zerlegung zu demonstrieren (Anmerkung: Ich möchte diese Zerlegung nur unter Verwendung von Tensoreigenschaften demonstrieren, nicht von Young-Tableaus). Ich habe versucht, nach Georgi, Hamermesh, Zee und irgendwo im Internet zu suchen, aber ich habe keine gute Referenz gefunden, die diese repräsentative Zerlegung gut erklärt ...

BEARBEITEN: Die Demonstration sollte nicht die Verwendung von Young-Diagrammen beinhalten ... mein Professor begann die Demonstration mit dem Schreiben ϵ ρ , ich , k Q ich S k , l = T ρ ' l = 8 M S und stoppte dann die Demonstration.

Verwandt S U ( 3 ) Beiträge: physical.stackexchange.com/q/156595/2451 , Physics.stackexchange.com /q/89173/2451 , Physics.StackExchange.com /q/147243/2451 , Physics.StackExchange.com /q/10403/2451 und Links darin; insbesondere die Antwort physical.stackexchange.com/a/14586/2451 und die darin enthaltenen Links.
@diracpaul: 1) Ich muss zugeben, ich verstehe Ihre Frustration darüber, dass Sie zB diesen Phys.SE-Beitrag nach so viel Arbeit in der Warteschleife sehen, aber dies ist eine Entscheidung der Community. 2) Viele von Ihnen S U ( 3 ) Posts ähnlich diesem Post sind noch offen, zB this und this . 3) Löschungen von HW-Antworten sind in der Regel nur vorübergehend. 4) Sprechen Sie über eine andere Ihrer Antworten? Welche?

Antworten (2)

Da diese Frage wie eine Hausaufgabe aussieht, fassen wir uns etwas kurz. Die Notizen von OP beschreiben anscheinend die Symmetrie des entsprechenden Young-Diagramms für jeden S U ( 3 ) irrep. Jedes Kästchen entspricht einem Index. Grob gesagt sind Indizes in derselben Zeile (Spalte) jeweils symmetrisch (antisymmetrisch).

Beispiele:

  1. Eine einzige Kiste [     ] entspricht der fundamentalen irrep 3 .

  2. Zwei Boxen übereinander [     ] [     ] ist die antifundamentale irrep 3 ¯ wenn wir mit Hilfe des Levi-Civita-Symbols dualisieren ϵ ich J k . Hier passen wir die Vorzeichenkonvention an ϵ 123 = 1 = ϵ 123 .

  3. Das Tensorprodukt 3 3 3 ¯ 6 S entspricht

    [     ] [ A ] [     ] [ A ] [     ] [ A ]

    oder T ich J = ϵ ich J k A k + S ich J , Wo A k := 1 2 T ich J ϵ ich J k .

  4. Das Tensorprodukt 3 ¯ 3 1 8 M entspricht

    [     ] [     ] [ A ] [     ] [     ] [ A ] [     ] [ A ] [     ]
    oder T ich J = S δ J ich + M ich J , Wo S := 1 3 T ich ich , Und T R M = 0 .

  5. Das Tensorprodukt 6 S 3 8 M 10 S entspricht

    [     ] [     ] [ A ] [     ] [     ] [ A ] [     ] [     ] [ A ]
    oder T ich J , k = { M ich ϵ J k + ( ich J ) } + S ich J k , Wo M ich := 1 3 T ich J , k ϵ J k , Und T R M = 0 .

Verweise:

  1. H. Georgi, Lie Algebras in Particle Physics, 1999, Abschnitt 13.2.

  2. JJ Sakurai, Moderne Quantenmechanik, 1994, Abschnitt 6.5.

Da die akzeptierte Antwort das Problem aus der Sicht von Young-Diagrammen behandelt und ich selbst einige Probleme mit Tensormethoden hatte, halte ich es für sinnvoll, hier den Tensoransatz hinzuzufügen.

Gemischt antisymmetrisch

Über die Bedeutung von M A , erinnere dich daran S U ( 3 ) Darstellungen können in Begriffen von "oben symmetrisierten Indizes" und "unten symmetrisierten Indizes" charakterisiert werden (siehe zum Beispiel den Anfang von Kapitel V.2 von Zee's Group Theory in a Nutshell for Physicists ). In anderen Welten, während ( 𝑇 𝑗 𝑖 1 3 𝛿 J ich 𝑇 k k ) nur zwei Indizes hat, könnten wir auch den unteren Index mit einem Levi-Civita-Symbol erhöhen und so erhalten ( 𝑇 𝑗 𝑖 ϵ J k l 1 3 ϵ ich k l 𝑇 k k ) , die nun in zwei Indizes explizit antisymmetrisch ist. Die Lektion hier ist, dass in S U ( 3 ) , dank der besonderen Existenz eines Levi-Civita-Symbols mit drei Indizes können wir jeden Tensor in Form von Tensoren mit symmetrischen Indizes schreiben, die diese „oben“ oder „unten“ sind.

Rechnen 3 6 S

Lassen Sie uns dann fortfahren, Tensormethoden zu verwenden, um das Tensorprodukt der Darstellungen zu schreiben. Wir haben hier einen Vektor Q ich Transformation in die fundamentale Darstellung, 3 , und ein symmetrischer Tensor S J k Verwandlung in die 6 S Darstellung. Wie Sie bemerkt haben, verwandelt sich ein Objekt in die 3 6 S Darstellung verwandelt sich als Q ich S J k . Da die Darstellungen von S U ( 3 ) durch symmetrische Tensoren mit Indizes oben oder unten gekennzeichnet sind, wird unser Ziel sein, die antisymetrischen Teile in jeder Indexkombination zu trennen.

Wir können die Indizes von nicht antisymmetrisieren S J k , da es sich um einen symmetrischen Tensor handelt. Es bleibt uns übrig, den Index von zu antisymmetrisieren Q ich mit jedem der Indizes von S J k . Daher verwenden wir das Levi-Civita-Symbol, um zu erhalten

Q ich S J k = 1 3 ϵ ich J l ϵ l M N Q M S N k + 1 3 ϵ ich k l ϵ l M N Q M S J l + ( Q ich S J k 1 3 ϵ ich J l ϵ l M N Q M S N k 1 3 ϵ ich k l ϵ l M N Q M S J l )

Die Koeffizienten werden so gewählt, dass der letzte Term in den drei Indizes vollständig symmetrisch ist. Eine andere Möglichkeit wäre den Ansatz zu picken S ich J k = Q ich S J k A ϵ ich J l ϵ l M N Q M S N k B ϵ ich k l ϵ l M N Q M S J l und die Bedingung stellen, dass S ich J k in allen Indizes symmetrisch ist, um die korrekten Koeffizienten zu erhalten.

Entweder, jetzt ist der erste Term symmetrisch unter J k und hat keine bestimmte Symmetrie, wenn ich gilt als. Der letzte Term entspricht einem vollsymmetrischen Tensor, der hat 10 unabhängige Komponenten, d.h. es ist a 10 S . Daher hat der erste Term 8 unabhängige Komponenten und entspricht folglich einer 8 M S , Die M S kommt von gemischter Symmetrie.

Verweise

  1. A. Zee, Gruppentheorie auf den Punkt gebracht für Physiker (Princeton University Press: Princeton, 2016). Kapitel V.2.